Difference between revisions of "2013 AMC 10B Problems/Problem 13"

(Redirected page to 2013 AMC 12B Problems/Problem 7)
 
(5 intermediate revisions by 3 users not shown)
Line 1: Line 1:
Jo and Blair take turns counting from <math>1</math> to one more than the last number said by the other person. Jo starts by saying "<math>1</math>", so Blair follows by saying "<math>1, 2</math>" . Jo then says "<math>1, 2, 3</math>" , and so on. What is the 53rd number said?
+
#REDIRECT [[2013 AMC 12B Problems/Problem 7]]
 
 
<math> \textbf{(A)}\ 2\qquad\textbf{(B)}\ 3\qquad\textbf{(C)}\ 5\qquad\textbf{(D)}\ 6\qquad\textbf{(E)}\ 8 </math>
 

Latest revision as of 12:11, 7 April 2013